Уніфікований PDF різниці двох оборотів


9

Чи можливо, щоб PDF різницю двох iid rv виглядав як прямокутник (замість, скажімо, трикутника, який ми отримуємо, якщо rv взяті з рівномірного розподілу).

тобто чи можливо для PDF f jk (для двох iid rv, взятих з деякого розподілу) f (x) = 0,5 для всіх -1 <x <1?

Існує ніяких обмежень на розподіл, з якого беремо j та k, за винятком того, що min дорівнює -1, а max 1.

Після деяких експериментів я думаю, що це може бути неможливим.


Різниця двох рівномірних розподілів - це трикутний розподіл, тому якщо ви запитаєте, чи можна отримати рівномірність різниці однорідних уніформ, то відповідь - ні.
Тім

Тут же запитують Q: math.stackexchange.com/questions/2048939/… поки що без відповідей!
kjetil b halvorsen

Дійсно, важко уникнути реалізацій поза коли і і мають масу ймовірностей, близьку до цих кінцевих точок. [1,1]jk
Крістоф Ганк

2
Це неможливо. На мій спогад, це (у дещо іншій формі) вже відповіло десь на місці. Я побачу, чи зможу його знайти
Glen_b -Встановіть Моніку

1
@Glen_b Ви можете згадати stats.stackexchange.com/questions/125360/… . Однак це не зовсім дублікат, оскільки різниця змінних iid, хоча виражається як сума може включати суму змінних з невідповідними розподілами. Я вірю, що тривіальна модифікація мого рішення дозволить вирішити цю різницю; Рішення Silverfish виглядає так, що воно застосовується безпосередньо майже без модифікацій, але спочатку потрібно видалити багато сторонніх матеріалів, щоб побачити це. XYX+(Y),
whuber

Відповіді:


10

Теорема: Не існує розподілу для якого коли .DistABU(1,1)A,BIID Dist


Доведення: Розглянемо дві випадкові величини із загальною характеристичною функцією . Позначимо їх різницю через . Характерна функція різниці:A,BIID DistφD=AB

φD(t)=E(exp(itD))=E(exp(it(AB)))=E(exp(itA))E(exp(itB))=φ(t)φ(t)=φ(t)φ(t)¯=|φ(t)|2.

(Четвертий рядок цього твору випливає з того, що характерною функцією є ерміціанка .) Тепер, беручи надає форму , яка:DU(1,1)φD

φD(t)=E(exp(itD))=Rexp(itr)fD(r)dr=1211exp(itr)dr=12[exp(itr)it]r=1r=1=12exp(it)exp(it)it=12(cos(t)+isin(t))(cos(t)+isin(t))it=12(cos(t)+isin(t))(cos(t)isin(t))it=122isin(t)it=sin(t)t=sinc(t).

де остання є (ненормалізованою) функцією sinc . Отже, щоб відповідати вимогам до , нам потрібна характерна функція з нормою квадрата, заданою:Distφ

|φ(t)|2=φD(t)=sinc(t).

Ліва частина цього рівняння є нормою квадрата і тому є негативною, тоді як права частина - це функція, яка в різних місцях є негативною. Отже, рішення цього рівняння не існує, і тому немає характерної функції, яка б відповідала вимогам розподілу. (Хат-підказка Фабіану для вказівки на це у спорідненому питанні щодо математики.SE .) Отже, розподілу з вимогами теореми немає.


3

Це питання інженера-електрика щодо цього питання, з точки зору, який більше підходить для dsp.SE, а не для статистики.SE, але не важливо.

Припустимо, що і - неперервні випадкові величини зі спільним pdf . Тоді, якщо позначає , маємо, що Нерівність Коші-Шварца говорить нам, що має максимум при . Насправді, оскільки насправді функція "автокореляції" розглядається як "сигнал", вона повинна мати унікальний максимум при і тому не може бути рівномірно розподілений, як бажано. Як варіант, якщоXYf(x)ZXY

fZ(z)=f(x)f(x+z) dx.
fZ(z)z=0fZfz=0Z fZнасправді були рівномірною щільністю (пам'ятайте, що це також функція автокореляції), тоді "спектральна щільність потужності" (розглядається як сигнал) була б функцією sinc, і, таким чином, не була негативною функцією, оскільки всі спектральні щільності потужності повинні бути . Ерго, припущення, що - рівномірна щільність, призводить до суперечності, і тому припущення повинно бути помилковим.fZfZ

Твердження, що , очевидно, недійсне, коли загальний розподіл та містить атоми, оскільки в такому випадку розподіл також буде містити атоми. Я підозрюю, що обмеження на те, що і мають pdf, може бути знято, і чисто загальнотеоретичний доказ побудований для загального випадку, коли і не обов'язково користуються файлом pdf (але їхня різниця є).fZU[1,1]XYZXYXY


1
Частина цього мені не здається правильною. Характеристична функція розподіл є функція, так ясно , що вид перетворення Фур'є є допустимим. Мені здається, ваша логіка веде до надмірного доказування - це, здається, доводить не тільки те, що не може бути рівномірним, але і рівномірний розподіл взагалі не може існувати. Я неправильно зрозумів? U(1,1)sincZ
Бен - Відновити Моніку

1
Чи існує характерна функція , це не проблема; вона існує. Pdf від - це функція автокореляції . Що ж, спектральна щільність потужності будь-якої функції автокореляції повинна бути негативною функцією. Отже, припущення, що призводить до спектральної щільності потужності, яка є функцією sinc (яка приймає як позитивні, так і негативні значення). Оскільки це не є дійсною спектральною щільністю потужності (пам’ятайте, що є функцією автокореляції), припущення, що має бути помилковим.U[1,1]ZfZU[1,1]fZfZU[1,1]
Діліп Сарват
Використовуючи наш веб-сайт, ви визнаєте, що прочитали та зрозуміли наші Політику щодо файлів cookie та Політику конфіденційності.
Licensed under cc by-sa 3.0 with attribution required.